Merge remote branch 'public/master'
[course.git] / latex / problems / Serway_and_Jewett_4 / problem03.09.tex
1 \begin{problem*}{3.9}
2 The mountain lion can jump to a height of $h = 12.0\U{ft}$ when
3 leaving the ground at an angle of $\theta = 45.0\dg$.  With what
4 speed, in SI units, does it leave the ground to make the leap?
5 \end{problem*} % problem 3.9
6
7 \begin{solution}
8 First, we'll convert the height into SI units:
9 \begin{equation}
10  h = 12.0\U{ft} \p[{ \frac{1\U{m}}{3.28\U{ft}} }]
11         = 3.659\U{m}
12 \end{equation}
13
14 Next, arrange the information we know in a table, calling the launch
15 point $P_0$ and the peak point $P_1$.\\
16 \begin{tabular}{r||r|r|}
17  Point & $P_0$ & $P_1$ \\
18  \hline
19  \hline
20  $a_x$ & \multicolumn{2}{|c|}{$0\U{m}$} \\
21  \hline
22  $a_y$ & \multicolumn{2}{|c|}{$-9.8\U{m}$} \\
23  \hline
24  $v_x$ & \multicolumn{2}{|c|}{?} \\
25  \hline
26  $v_y$ & ? & $0\U{m/s}$ \\
27  \hline
28  $x$   & $0\U{m}$ & ? \\
29  \hline
30  $y$   & $0\U{m}$ & $3.66\U{m}$ \\
31  \hline
32  $t$   & $0\U{s}$ & ? \\
33  \hline
34 \end{tabular}\\
35 Where we know $v_1 = 0\U{m/s}$ because $P_1$ is at the apex of the jump
36  and $x_0$, $y_0$, and $t_0$ through our choice of coordinate frame.
37
38 We want to pick an equation to tell us something about the initial velocity
39  (because they told us $\theta$, either $v_{x0}$ or $v_{y0}$ will suffice.).
40 Looking at our 4 constant acceleration equations (text p. 53),
41 \begin{align}
42  v_{xf} &= v_{xi} + a_x t \\
43  x_f &= x_i + \frac{1}{2}(v_{xf} + v_{xi}) t \\
44  x_f &= x_i + v_{xi} t + \frac{1}{2} a_x t^2 \label{eqn.t_sqr}\\
45  v_{xf}^2 &= v_{xi}^2 + 2 a_x (x_f - x_i) \label{eqn.v_sqr}
46 \end{align}
47 We see that eqn.~\ref{eqn.v_sqr} applied to the $y$ direction is
48 perfect, because it has no information in it that we don't already
49 know except $v_{y0}$.
50 \begin{align}
51  v_{y1}^2 &= (0\U{m/s})^2 = v_{y0}^2 + 2 a_y (y_1 - y_0) \\
52  v_{y0}^2 &= -2 a_y y_1 \\
53  v_{y0} &= \sqrt{ -2 a_y y_1 }
54 \end{align}
55 And we use the angle to solve for the magnitude of the inital velocity:
56 \begin{align}
57  v_{y0} &= v_0 \sin \theta \\
58  v_0 &= \frac{v_{y0}}{\sin \theta}
59         = \frac{\sqrt{ -2 a_y y_1 }}{\sin \theta}
60         = \frac{\sqrt{-2 \cdot (-9.8\U{m/s}^2) \cdot 3.66\U{m}}}{\sin 45^o}
61         = \ans{12.0\U{m/s}}
62 \end{align}
63 \end{solution}